Những câu hỏi liên quan
Văn Quyết
Xem chi tiết
hiền nguyễn
Xem chi tiết
Nguyễn Lê Phước Thịnh
19 tháng 4 2023 lúc 10:59

\(\dfrac{\sqrt{1+x^3+y^3}}{xy}>=\sqrt{\dfrac{3}{xy}}\)

\(\dfrac{\sqrt{1+y^3+z^3}}{yz}>=\sqrt{\dfrac{3}{yz}}\)

\(\dfrac{\sqrt{1+z^3+x^3}}{xz}>=\sqrt{\dfrac{3}{xz}}\)

=>\(VT>=\sqrt{3}\left(\dfrac{1}{\sqrt{xy}}+\dfrac{1}{\sqrt{yz}}+\dfrac{1}{\sqrt{xz}}\right)=3\sqrt{3}\)

Bình luận (0)
Đỗ Thị Ánh Nguyệt
Xem chi tiết
bac luu
Xem chi tiết
Adu vip
Xem chi tiết
Nguyễn Huy Tú ( ✎﹏IDΣΛ...
14 tháng 7 2021 lúc 9:19

undefined

Bình luận (0)
An Thy
14 tháng 7 2021 lúc 9:21

\(\dfrac{1}{x}+\dfrac{1}{y}+\dfrac{1}{z}\ge\dfrac{1}{\sqrt{xy}}+\dfrac{1}{\sqrt{yz}}+\dfrac{1}{\sqrt{zx}}\)

\(\Rightarrow\dfrac{2}{x}+\dfrac{2}{y}+\dfrac{2}{z}\ge\dfrac{2}{\sqrt{xy}}+\dfrac{2}{\sqrt{yz}}+\dfrac{2}{\sqrt{zx}}\)

\(\Rightarrow\dfrac{2}{x}+\dfrac{2}{y}+\dfrac{2}{z}-\dfrac{2}{\sqrt{xy}}+\dfrac{2}{\sqrt{yz}}+\dfrac{2}{\sqrt{zx}}\ge0\)

\(\Rightarrow\dfrac{1}{x}-\dfrac{2}{\sqrt{xy}}+\dfrac{1}{y}+\dfrac{1}{y}-\dfrac{2}{\sqrt{yz}}+\dfrac{1}{z}+\dfrac{1}{z}-\dfrac{2}{\sqrt{zx}}+\dfrac{1}{x}\ge0\)

\(\Rightarrow\left(\dfrac{1}{\sqrt{x}}-\dfrac{1}{\sqrt{y}}\right)^2+\left(\dfrac{1}{\sqrt{y}}-\dfrac{1}{\sqrt{z}}\right)^2+\left(\dfrac{1}{\sqrt{z}}-\dfrac{1}{\sqrt{x}}\right)^2\ge0\) (luôn đúng)

Dấu = xảy ra khi \(x=y=z\)

Bình luận (0)
Dưa Hấu
14 tháng 7 2021 lúc 9:21

undefined

Bình luận (0)
Thiều Khánh Vi
Xem chi tiết
Akai Haruma
25 tháng 9 2018 lúc 23:33

Lời giải:

Đặt \((\sqrt{x}, \sqrt{y}, \sqrt{z})=(a,b,c)\Rightarrow abc=1\)

Bài toán trở thành chứng minh:

\(\frac{1}{(ab+a+1)^2}+\frac{1}{(bc+b+1)^2}+\frac{1}{(ca+c+1)^2}\geq \frac{1}{3}\)

------------

Áp dụng 1 kết quả quen thuộc của BĐT AM-GM: \(x^2+y^2+z^2\geq \frac{(x+y+z)^2}{3}\) ta có:

\(\frac{1}{(ab+a+1)^2}+\frac{1}{(bc+b+1)^2}+\frac{1}{(ca+c+1)^2}\geq \frac{1}{3}\left(\frac{1}{ab+a+1}+\frac{1}{bc+b+1}+\frac{1}{ca+c+1}\right)^2\)

Mà:

\(\frac{1}{ab+a+1}+\frac{1}{bc+b+1}+\frac{1}{ca+c+1}=\frac{c}{abc+ac+c}+\frac{ac}{bc.ac+b.ac+ac}+\frac{1}{ac+c+1}\)

\(=\frac{c}{1+ac+c}+\frac{ac}{c+1+ac}+\frac{1}{ac+c+1}=\frac{ac+c+1}{ac+c+1}=1\) (thay $abc=1$)

Do đó:

\(\frac{1}{(ab+a+1)^2}+\frac{1}{(bc+b+1)^2}+\frac{1}{(ca+c+1)^2}\geq \frac{1}{3}.1^2=\frac{1}{3}\) (đpcm)

Dâu bằng xảy ra khi $a=b=c=1$ hay $x=y=z=1$

Bình luận (0)
Big City Boy
Xem chi tiết
Lấp La Lấp Lánh
31 tháng 10 2021 lúc 12:05

ĐKXĐ: \(\left\{{}\begin{matrix}x\ge0\\y\ge0\\z\ge0\end{matrix}\right.\)

\(A=\dfrac{\sqrt{x}}{\sqrt{xy}+\sqrt{x}+2}+\dfrac{\sqrt{y}}{\sqrt{yz}+\sqrt{y}+1}+\dfrac{2\sqrt{z}}{\sqrt{xz}+2\sqrt{z}+2}\)

\(=\dfrac{\sqrt{x}}{\sqrt{xy}+\sqrt{x}+\sqrt{xyz}}+\dfrac{\sqrt{y}}{\sqrt{yz}+\sqrt{y}+1}+\dfrac{\sqrt{xyz}.\sqrt{z}}{\sqrt{xz}+\sqrt{xyz}.\sqrt{z}+\sqrt{xyz}}\)

\(=\dfrac{1}{\sqrt{yz}+\sqrt{y}+1}+\dfrac{\sqrt{y}}{\sqrt{yz}+\sqrt{y}+1}+\dfrac{\sqrt{yz}}{\sqrt{yz}+\sqrt{y}+1}\)

\(=\dfrac{\sqrt{yz}+\sqrt{y}+1}{\sqrt{yz}+\sqrt{y}+1}=1\)

\(\Rightarrow\sqrt{A}=\sqrt{1}=1\)

Bình luận (0)
Kimian Hajan Ruventaren
Xem chi tiết
Nguyễn Việt Lâm
16 tháng 1 2021 lúc 20:47

\(P\ge\dfrac{\sqrt{3\sqrt[3]{x^3y^3}}}{xy}+\dfrac{\sqrt{3\sqrt[3]{y^3z^3}}}{yz}+\dfrac{\sqrt{3\sqrt[3]{z^3x^3}}}{zx}\)

\(P\ge\sqrt{3}\left(\dfrac{1}{\sqrt{xy}}+\dfrac{1}{\sqrt{yz}}+\dfrac{1}{\sqrt{zx}}\right)\ge\sqrt{3}.3\sqrt[3]{\dfrac{1}{\sqrt{xy.yz.zx}}}=3\sqrt{3}\)

Dấu "=" xảy ra khi \(x=y=z=1\)

Bình luận (0)
tthnew
16 tháng 1 2021 lúc 20:50

Ta có bất đẳng thức sau \(x^3+y^3\ge xy\left(x+y\right)\Leftrightarrow\left(x+y\right)\left(x-y\right)^2\ge0.\)

Do đó:

\(P=\sum\dfrac{\sqrt{1+x^3+y^3}}{xy}\ge\sum\dfrac{\sqrt{xyz+xy\left(x+y\right)}}{xy}\)

\(=\sqrt{x+y+z}\left(\dfrac{1}{\sqrt{xy}}+\dfrac{1}{\sqrt{yz}}+\dfrac{1}{\sqrt{zx}}\right)\ge\sqrt{3\sqrt[3]{xyz}}\cdot3\sqrt[3]{\dfrac{1}{\sqrt{xy}}\cdot\dfrac{1}{\sqrt{yz}}\cdot\dfrac{1}{\sqrt{zx}}}=3\sqrt{3}\)

Đẳng thức xảy ra khi $x=y=z=1.$

Bình luận (0)
Hồng Phúc
16 tháng 1 2021 lúc 20:51

Áp dụng BĐT AM-GM:

\(P=\dfrac{\sqrt{1+x^3+y^3}}{xy}+\dfrac{\sqrt{1+y^3+z^3}}{yz}+\dfrac{\sqrt{1+z^3+x^3}}{zx}\)

\(\ge\dfrac{\sqrt{3xy}}{xy}+\dfrac{\sqrt{3yz}}{yz}+\dfrac{\sqrt{3zx}}{zx}\)

\(=\sqrt{3}\left(\dfrac{1}{\sqrt{xy}}+\dfrac{1}{\sqrt{yz}}+\dfrac{1}{\sqrt{zx}}\right)\)

\(\ge\sqrt{3}.3\sqrt[3]{\dfrac{1}{xyz}}=3\sqrt{3}\)

\(minP=3\sqrt{3}\Leftrightarrow x=y=z\)

Bình luận (0)
Nguyễn Hoàng Minh
Xem chi tiết
Nguyễn Việt Lâm
26 tháng 3 2022 lúc 15:39

\(\sqrt{x+yz}=\sqrt{x\left(x+y+z\right)+yz}=\sqrt{\left(x+y\right)\left(z+x\right)}\ge\sqrt{\left(\sqrt{xz}+\sqrt{xy}\right)^2}=\sqrt{xy}+\sqrt{xz}\)

\(\Rightarrow\dfrac{x}{x+\sqrt{x+yz}}\le\dfrac{x}{x+\sqrt{xy}+\sqrt{xz}}=\dfrac{\sqrt{x}}{\sqrt{x}+\sqrt{y}+\sqrt{z}}\)

Tương tự:

\(\dfrac{y}{y+\sqrt{y+xz}}\le\dfrac{\sqrt{y}}{\sqrt{x}+\sqrt{y}+\sqrt{z}}\)

\(\dfrac{z}{z+\sqrt{z+xy}}\le\dfrac{\sqrt{z}}{\sqrt{x}+\sqrt{y}+\sqrt{z}}\)

Cộng vế:

\(VT\le\dfrac{\sqrt{x}+\sqrt{y}+\sqrt{z}}{\sqrt{x}+\sqrt{y}+\sqrt{z}}=1\)

Dấu "=" xảy ra khi \(x=y=z=\dfrac{1}{3}\)

Bình luận (0)